相似三角形

extbf{平行线分线段成比例定理:}两条直线被一组平行线所截,所得的对应线段成比例,即:若直线$l_1parallel l_2parallel l_3$,则有
[frac{AB}{BC}=frac{DE}{EF},qquad frac{AB}{AC}=frac{DE}{DF},qquad frac{BC}{AC}=frac{EF}{DF}.]

对应角相等、对应边成比例的三角形,叫做相似三角形.

相似三角形对应边的比,叫做相似比(或相似系数).

判定三角形相似的方法,有

定理1.平行于三角形一边的直线和其他两边(或两边的延长线)相交,所构成的三角形与原三角形相似.

定理2.如果一个直角三角形的斜边和一条直角边与另一个直角三角形的斜边和一条直角边对应成比例,那么这两个三角形相似.

判定定理1.如果一个三角形的两个角与另一个三角形的两个角对应相等,那么这两个三角形相似.

判定定理2.如果一个三角形的两条边与另一个三角形的两条边对应成比例,且对应夹角相等,那么这两个三角形相似.

判定定理3.如果一个三角形的三条边与另一个三角形的三条边对应成比例,那么这两个三角形相似.

相似三角形有如下重要性质:

(1)相似三角形的对应角相等,对应边成比例.

(2)相似三角形的对应边上的重要线段(如中线、高等)成比例,且等于相似比;对应角的平分线成比例,且等于相似比.

(3)相似三角形的面积的比等于相似比的平方.

1.在$ riangle ABC$中, $angle ABC=60^circ$,点$P$是$ riangle ABC$内的一点,使得 $angle APB=angle BPC=angle CPA$,且$PA=8,PC=6$,求$PB$.

2.已知$ riangle ABC$,向外作正方形$ABPQ$和正方形$ACMN$.若$BCparallel PM$,求证: $AB=AC$.

3.证明角平分线定理:

(1) $ riangle ABC$中,若$AP$是$angle A$的平分线,则$frac{BP}{PC}=frac{AB}{AC}$.

(2)外角平分线定理: $ riangle ABC$中,若$AQ$是$angle A$的外平分线,则$frac{BQ}{QC}=frac{AB}{AC}$.

4.证明 extbf{梅涅劳斯定理}:设$A'$、$B'$、$C'$分别是$ riangle ABC$的三边$BC$、$CA$、$AB$或其延长线上的点,若$A'$、$B'$、$C'$三点在一条直线上,则
[frac{BA'}{A'C}cdot frac{CB'}{B'A}cdot frac{AC'}{C'B}=1.]

提示:过$A$作$ADparallel C'A'$.

5.重心定理:三角形顶点到重心的距离等于该顶点对边上中线长的$frac{2}{3}$. (注:重心是三角形三条中线的交点)

6.已知$ riangle ABC$中, $O$是三角形内一点满足: $angle BAO= angle CAO=angle CBO=angle ACO$.求证: $ riangle ABC$三边长成等比数列,即$BC^2=ACcdot AB$. %(2010年北大保送生考试数学试题)

提示:过$O$作$AC$的平行线交$BC,AB$于$D,E$,则$ riangle AOEsim riangle ACO$和$ riangle OCDsim riangle BCD$.

7.如图,在$ riangle ABC$中,已知$CD$为边$AB$上的高,正方形$EFGH$的四个顶点分别在$ riangle ABC$上,求证: $frac{1}{AB}+ frac{1}{CD}=frac{1}{EF}$.

8.如图,已知一个三角形纸片$ABC$, $BC$边的长为$8$, $BC$边上的高为$6$, $angle B$和$angle C$都为锐角, $M$为$AB$一动点(点$M$与点$A$、$B$不重合),过点$M$作$MNparallel BC$,交$AC$于点$N$,在$ riangle AMN$中,设$MN$的长为$x$, $MN$上的高为$h$.

(1)请你用含$x$的代数式表示$h$.

(2)将$ riangle AMN$沿$MN$折叠,使$ riangle AMN$落在四边形$BCNM$所在平面,设点$A$落在平面的点为$A_1$, $ riangle A_1MN$与四边形$BCNM$重叠部分的面积为$y$,当$x$为何值时, $y$最大,最大值为多少?

9.如图所示, $ riangle ABC$和$ riangle DEF$均为直角三角形, $angle ACB=angle DFE=90^circ$且$AC=4,BC=2,DF=EF=4$.现已知阴影部分的面积为$frac{5}{2}$,求$BF$的长度.

高考数学压轴题分析

介绍导数压轴题的重要方法和技巧,学会利用参变分离,二阶导,极值点偏移和隐零点等方法解决参数取值范围,函数不等式以及最值问题;介绍圆锥曲线压轴题中韦达公式和弦长定理的运用,灵活处理直径,定点,参数取值范围和最值问题,给高考尖子生提供高分秘诀。


2020北京新高考高三学生或高一高二尖子生

帮助北京高三学生数学赢得高分


section{Algebra}

subsection{Inequality}

egin{problem}
Find the value of $x+y$ for which the expression
[frac{6 x^{2}}{y^{6}}+frac{6 y^{2}}{x^{6}}+9 x^{2} y^{2}+frac{4}{x^{6} y^{6}}]
is minimized.
end{problem}
egin{solution}
利用均值不等式
egin{align*}
frac{6 x^{2}}{y^{6}}+frac{6 y^{2}}{x^{6}}+9 x^{2} y^{2}+frac{4}{x^{6} y^{6}} \
=left(frac{6 x^{2}}{y^{6}}+frac{6 y^{2}}{x^{6}}+3 x^{2} y^{2}+3 x^{2} y^{2} ight)+left(x^{2} y^{2}+x^{2} y^{2}+x^{2} y^{2}+frac{4}{x^{6} y^{6}} ight) \
geq 4 sqrt[4]{frac{6 x^{2}}{y^{6}} cdot frac{6 y^{2}}{x^{6}} cdot 3 x^{2} y^{2} cdot 3 x^{2} y^{2}}+4 sqrt[4]{x^{2} y^{2} cdot x^{2} y^{2} cdot x^{2} y^{2} cdot frac{4}{x^{6} y^{6}}}=16 sqrt[4]{4}.
end{align*}
当且仅当$frac{6 y^{2}}{x^{6}}=3 x^{2} y^{2}, x^{2} y^{2}=frac{4}{x^{6} y^{6}}$,即$x^8=y^8=2$时取等成立,此时有$x+y=-2sqrt[8]{2},0$或$2sqrt[8]{2}$.
end{solution}


egin{problem}
If $a,b,c$, and $d$ are reals with $ageq bgeq cgeq dgeq 0$ and $b(b-a)+c(c-b)+d(d-c) leq 2-frac{a^{2}}{2}$, find
the minimum value of the expression
[frac{1}{b+2006 c-2006 d}+frac{1}{a+2006 b-2006 c-d}+frac{1}{2007 a-2006 b-c+d}+frac{1}{a-b+c+2006 d}.]
end{problem}
egin{solution}
设$d=x,c=x+y,b=x+y+z,a=x+y+z+w$.因为$ageq bgeq cgeq dgeq 0$,故$x,y,z,wgeq 0$.因为$b(b-a)+c(c-b)+d(d-c) leq 2-frac{a^{2}}{2}$,所以
[(x+y+z)(-w)+(x+y)(-z)+x(-y) leq 2-frac{(x+y+z+w)^{2}}{2},]
即[x^2+y^2+z^2+w^2leq 4.]
由均值不等式可知
[frac{x+y+z+w}{4} leq sqrt{frac{x^{2}+y^{2}+z^{2}+w^{2}}{4}} leq 1,]
即$x+y+z+wleq 4$.
由Cauchy不等式可知,原不等式等价于
egin{align*}
&frac{1}{x+2007 y+z}+frac{1}{y+2007 z+w}+frac{1}{z+2007 w+x}+frac{1}{w+2007 x+y} \
&geq frac{(1+1+1+1)^{2}}{2009(x+y+z+w)} geq frac{16}{2009 imes 4}=frac{4}{2009},
end{align*}
当且仅当$x=y=z=w=1$时取等成立,此时$d=1,c=2,b=3,a=4$时,最小值为$frac{4}{2009}$.
end{solution}

subsection{Equation}

egin{problem}
If $a, b$, and $c$ are real numbers such that $a + b + c = 6$ and $ab + bc + ca = 9$,find the sum of all possible values of the expression $lfloor a floor+lfloor b floor+lfloor c floor$.
end{problem}

egin{problem}
For how many rational numbers p is the area of the triangle formed by the intercepts and vertex of $f(x) =-x^2 +4px-�p + 1$ an integer?
end{problem}

egin{problem}
Find the all values of $a$ such that $x^6-6x^5+12x^4+ax^3+12x^2-6x+1$ is nonnegative for all real $x$.
end{problem}
egin{solution}
分离变量法.当$x>0$时,有
[a geq-frac{x^{6}-6 x^{5}+12 x^{4}+12 x^{2}-6 x+1}{x^{3}}=-x^{3}-frac{1}{x^{3}}+6 x^{2}+frac{6}{x^{2}}-12 x-frac{12}{x},]
令$y=x+1/x$,有$y^3=x^{3}+frac{1}{x^{3}}+3y$且$y^2=x^{2}+frac{1}{x^{2}}+2$,因此
[-x^{3}-frac{1}{x^{3}}+6 x^{2}+frac{6}{x^{2}}-12 x-frac{12}{x}=-y^3+6y^2-9y-12=-y(y-3)^2-12]


end{solution}

egin{problem}


end{problem}
egin{solution}

end{solution}

egin{problem}
Find the real root of $x^{5}+5 x^{3}+5 x-1$.
end{problem}
egin{solution}
令$x=u-1/u$,得到$u^{5}-frac{1}{u^{5}}-1$,即$u^{10}-u^5-1=0$.
于是
[x=sqrt[5]{frac{sqrt{5}+1}{2}}-sqrt[5]{frac{sqrt{5}-1}{2}}.]
end{solution}

subsection{Sequence}


egin{solution}

end{solution}

egin{problem}
Given two sequences $x_n$ and $y_n$ defined by $x_0=y_0=7$,
egin{align*}{c}
x_{n} &=4 x_{n-1}+3 y_{n-1}, ext { and } \
y_{n} &=3 y_{n-1}+2 x_{n-1}
end{align*}
Find $lim _{n ightarrow infty} frac{x_{n}}{y_{n}}$.
end{problem}

比较如下两个数的大小: $left(1+frac{2}{3^{3}} ight)left(1+frac{2}{5^{3}} ight) cdotsleft(1+frac{2}{2013^{3}} ight)$与$sqrt{frac{3}{2}}$.

egin{problem}

[mathrm{M}=frac{1} {2018}+frac{1}{2019}+frac{1}{2020}+cdots+frac{1}{2050},]
求$frac{1}{M}$的整数部分.
end{problem}

egin{problem}
现有$10$个边长分别为$3,5,6,11,17,19,22,23,24,25$的正方形.试问:用这些正方形能否拼接(不许重叠,不许中空)成一个长方形?如果能,就给出这个长方形的长和宽,并请画出拼接图.如果不能,也请说明理由.
end{problem}

egin{problem}
(多选题)已知$f(x)=a x+ln x-frac{x^{2}}{x-ln x}$有三个互不相同的零点$x_1,x_2,x_3$且$x_1<x_2<x_3,g(x)=frac{x^2}{ax+ln x}$,则下列说法中正确的是(quad)

A. $a$的取值围是$1<a<frac{e}{e-1}-frac{1}{e}$

B. 若对任意的符合题干的$a$及$x_2,x_3$均有$x_2x_3^lambda<e^{lambda+1}$恒成立,则正实数$lambda$的取值范围是$lambdageq 1$

C. 对任意的符合题干的$x_1,x_2,x_3$均有
$left[frac{gleft(x_{1} ight)}{x_{1}} ight]^{2} cdot frac{gleft(x_{2} ight)}{x_{2}} cdot frac{gleft(x_{3} ight)}{x_{3}}>1$

D. 对任意的符合题干的$x_1,x_2,x_3$均有
$left[frac{gleft(x_{1} ight)}{x_{1}} ight]^{2} cdot frac{gleft(x_{2} ight)}{x_{2}} cdot frac{gleft(x_{3} ight)}{x_{3}}<1$
end{problem}

egin{problem}
正项数列${x_n}$满足$x_{n+1}=ln frac{e^{x_{n}}-1}{x_{n}}, x_{1}=1$.证明:
[frac{5}{12}<frac{frac{1}{x_{n}}-1}{2^{n}-2} leq frac{frac{1}{ln (e-1)}-1}{2}.]
end{problem}


egin{problem}
数列${a_n}$满足$a_{0}=d, a_{n+1}=lambda a_{n}^{3}-3 a_{n}, n=0,1,2, cdots, lambda>0, sqrt{lambda} d>2$.求通项公式.
end{problem}
egin{solution}
联想等式$x^{3}+frac{1}{x^{3}}+3left(x+frac{1}{x} ight)=left(x+frac{1}{x} ight)^{3}$,
设$sqrt{lambda} a_{n}=b_{n}, b_{0}=sqrt{lambda} d=mu+frac{1}{mu}$.
end{solution}


egin{problem}
数列${a_n}$满足$a_{n+1}=a_{n}^{2}-2, n=0,1,2, cdots$.求通项公式.
end{problem}
egin{solution}
$cos 2 heta$和$x+frac{1}{x}$.
end{solution}


section{导数}

egin{problem}
计算积分$int_{0}^{pi} frac{sin (2 n+1) x}{sin x} d x$,其中$n$为整数.
end{problem}
egin{solution}
egin{align*}
I&=int_{0}^{pi} frac{sin (2 n+1) x}{sin x} mathrm{d} x=int_{0}^{pi} 1+2 cos 2 x+cdots+2 cos 2 n x mathrm{d} x \
&=1+sin 2 x+cdots+frac{1}{n} sin 2 nleft.x ight|_{0} ^{pi} =pi.
end{align*}
end{solution}
事实上,我们有
egin{align*}
frac{sin 2 n x}{sin x} &=2 sum_{k=1}^{n} cos (2 k-1)x, \
frac{sin (2 n+1) x}{sin x} &=1+2 sum_{k=1}^{n} cos 2kx.
end{align*}


egin{problem}
求证Fejer积分$int_{0}^{frac{pi}{2}} left(frac{sin nx}{sin x} ight)^2 d x=frac{npi}{2}$,其中$n$为整数.
end{problem}
egin{solution}
事实上,
[I=int_{0}^{frac{pi}{2}} left(frac{sin nx}{sin x} ight)^2 d x=frac{1}{4}int_{-pi}^{pi} left(frac{sin nx}{sin x} ight)^2 d x.]
若$n=2k+1$,则$I=frac{1}{4}(2pi +4kpi)=frac{pi}{2}(2k+1)=frac{npi}{2}$.

若$n=2k$,则$I=frac{1}{4}cdot 4kpi=frac{pi}{2}cdot 2k=frac{npi}{2}$.
end{solution}


egin{problem}
(2005年高考全国1卷)
egin{enumerate}
item[(1)] 设函数$f(x)=x log _{2} x+(1-x) log _{2}(1-x)\,(0<x<1)$,求$f(x)$的最小值;

item[(2)] 设正数$p_1,p_2,p_3,cdots,p_{2^n}$满足$p_{1}+p_{2}+p_{3}+cdots+p_{2^{n}}=1$,证明:
[p_{1} log _{2} p_{1}+p_{2} log _{2} p_{2}+p_{3} log _{2} p_{3}+cdots+p_{2^{n}} log _{2} p_{2^{n}} geq -n.]

item[(3)] (熵不等式)设 $p_1,p_2,p_3,cdots,p_n$和 $q_1,q_2,q_3,cdots,q_n$均为正数且$sum_{k=1}^{n}p_k=sum_{k=1}^{n}q_k$.求证:
[sum_{k=1}^{n}p_kln p_kgeq sum_{k=1}^{n}p_kln q_k.]
end{enumerate}
end{problem}
egin{solution}
egin{enumerate}
item[(1)] 对函数$f(x)$求导数:
egin{align*}
f'(x) &=left(x log _{2} x ight)'+left[(1-x) log _{2}(1-x) ight]' \
&=log _{2} x-log _{2}(1-x)+frac{1}{ln 2}-frac{1}{ln 2} \ &=log _{2} x-log _{2}(1-x).
end{align*}
于是$f'left(frac{1}{2} ight)=0$.

当$x<frac{1}{2}$时, $f'(x)=log _{2} x-log _{2}(1-x)<0$, $f(x)$在区间$left(0,frac{1}{2} ight)$是减函数;当$x>frac{1}{2}$时, $f'(x)=log _{2} x-log _{2}(1-x)>0$, $f(x)$在区间$left(frac{1}{2},1 ight)$是增函数.

所以$f(x)$在$x=frac{1}{2}$时取得最小值, $fleft(frac{1}{2} ight)=-1$.

item[(2)] extbf{证法一:}用数学归纳法证明.

egin{enumerate}
item[(i)] 当$n=1$时,由(I)知命题成立.

item[(ii)] 假定当$n=k$时命题成立,即若正数$p_1,p_2,p_3,cdots,p_{2^k}$满足$p_{1}+p_{2}+p_{3}+cdots+p_{2^{k}}=1$,则
[p_{1} log _{2} p_{1}+p_{2} log _{2} p_{2}+p_{3} log _{2} p_{3}+cdots+p_{2^{k}} log _{2} p_{2^{k}} geq -k.]

当$n=k+1$时,若正数$p_1,p_2,p_3,cdots,p_{2^{k+1}}$满足$p_{1}+p_{2}+p_{3}+cdots+p_{2^{k+1}}=1$,令
[x=p_{1}+p_{2}+cdots+p_{2^{k}}, q_{1}=frac{p_{1}}{x}, q_{2}=frac{p_{2}}{x}, cdots, q_{2^{k}}=frac{p_{2^k}}{x}.]
则$q_1,q_2,q_3,cdots,q_{2^k}$为正数,且$q_{1}+q_{2}+cdots+q_{2^{k}}=1$.

由归纳假定知$q_{1} log _{2} q_{1}+q_{2} log _{2} q_{2}+cdots+q_{2^k} log _{2} q_{2^k} geq -k$.
egin{align*}
p_{1} log _{2} p_{1}+p_{2} log _{2} p_{2}+cdots+p_{2^k} log _2 p_{2^{k}} &=xleft(q_{1} log _{2} q_{1}+q_{2} log _{2} q_{2}+cdots+q_{2^k} log _{2} q_{2^k}+log _{2} x ight)\
&geq x(-k)+xlog_2 x, ag*{(1)}
end{align*}
同理,由$p_{2^{k}+1}+p_{2^{k}+2}+cdots+p_{2^{k+1}}=1-x$,可得
[p_{2^{k}+1} log _{2^{k}+1}+cdots+p_{2^{k+1}} log _{2^{k}+1}geq (1-x)(-k)+(1-x). log _{2}(1-x). ag*{(2)}]
综合(1)、(2)两式
egin{align*}
p_{1} log _{2} p_{1}+& p_{2} log _{2} p_{2}+cdots+p_{2^{k+1}} log _{2} p_{2^{k+1}} \
& geq [x+(1-x)](-k)+x log _{2} x+(1-x) log _{2}(1-x) \
& geq -(k+1).
end{align*}
即当$n=k+1$时命题也成立.
end{enumerate}
根据(i)、(ii)可知对一切正整数$n$命题成立.

extbf{证法二:}令函数$g(x)=x log _{2} x+(c-x) log _{2}(c-x)$ (常数$c>0,xin (0,c)$),那么
[g(x)=cleft[frac{x}{c} log _{2} frac{x}{c}+left(1-frac{x}{c} ight) log _{2}left(1-frac{x}{c} ight)+log _{2} c ight],]
利用(I)知,当$frac{x}{c}=frac{1}{2}$ (即$x=frac{c}{2}$)时,函数$g(x)$取得最小值.

于是对任意$x_1>0,x_2>0$,都有
egin{align*}
x_{1} log _{2} x_{1}+x_{2} log _{2} x_{2} & geq 2 cdot frac{x_{1}+x_{2}}{2} log _{2} frac{x_{1}+x_{2}}{2} \ &=left(x_{1}+x_{2} ight)left[log _{2}left(x_{1}+x_{2} ight)-1 ight]. ag*{(1)}
end{align*}

下面用数学归纳法证明结论.

egin{enumerate}
item[(i)] 当$n=1$时,由(I)知命题成立.

item[(ii)] 假定当$n=k$时命题成立,即若正数$p_1,p_2,p_3,cdots,p_{2^k}$满足$p_{1}+p_{2}+p_{3}+cdots+p_{2^{k}}=1$,有
[p_{1} log _{2} p_{1}+p_{2} log _{2} p_{2}+cdots+p_{2^{k}} log _{2} p_{2^{k}} geq -k.]

当$n=k+1$时, $p_1,p_2,p_3,cdots,p_{2^{k+1}}$满足$p_{1}+p_{2}+p_{3}+cdots+p_{2^{k+1}}=1$,令
[H=p_{1} log _{2} p_{1}+p_{2} log _{2} p_{2}+cdots+p_{2^{k+1}-1} log _{2} p_{2^{k+1}-1}+p_{2^{k+1}} log _{2} p_{2^{k+1}},]
由(1)得到
[H geqleft(p_{1}+p_{2} ight)left[log _{2}left(p_{1}+p_{2} ight)-1 ight]+cdots+left(p_{2^{k+1}-1}+p_{2^{k+1}} ight)left[log _{2}left(p_{2^{k+1}-1}+p_{2^{k+1}} ight)-1 ight],]
因为$left(p_{1}+p_{2} ight)+cdots+left(p_{2^{k+1}-1}+p_{2^{k+1}} ight)=1$,由归纳法假设
[left(p_{1}+p_{2} ight) log _{2}left(p_{1}+p_{2} ight)+cdots+left(p_{2^{k+1}-1}+p_{2^{k+1}} ight) log _{2}left(p_{2^{k+1-1}}+p_{2^{k+1}} ight) geq -k,]
得到
[H geq -k-left(p_{1}+p_{2}+cdots+p_{2^{k+1}-1}+p_{2^{k+1}} ight)=-(k+1).]
即当$n=k+1$时命题也成立.
end{enumerate}
所以对一切正整数$n$命题成立.

item[(3)] 利用不等式$tln tgeq t-1,t>0$.事实上,
egin{align*}
sum_{k=1}^{n} p_{k} ln p_{k}-sum_{k=1}^{n} p_{k} ln q_{k} &=sum_{k=1}^{n} p_{k} ln frac{p_{k}}{q_{k}
=sum_{k=1}^{n} q_{k} frac{p_{k}}{q_{k}} ln frac{p_{k}}{q_{k}}} \
&geq sum_{k=1}^{n} q_{k}left(frac{p_{k}}{q_{k}}-1 ight)=sum_{k=1}^{n}left(p_{k}-q_{k} ight)=0.
end{align*}
end{enumerate}
end{solution}

egin{problem}
(2004年高考全国2卷)已知函数$f(x)=ln (1+x)-x, g(x)=x ln x$.
egin{enumerate}
item[(1)] 求函数$f(x)$的最大值;

item[(2)] 设$0<a<b$,证明: $0<g(a)+g(b)-2 gleft(frac{a+b}{2} ight)<(b-a) ln 2$.
end{enumerate}
end{problem}
egin{solution}
egin{enumerate}
item[(1)] 函数$f(x)$的定义域是$(-1,infty),f'(x)=frac{1}{1+x}-1$.令$f'(x)=0$,解得$x=0$.当$-1<x<0$时, $f'(x)>0$;当$x>0$时, $f'(x)<0$,又$f(0)=0$,故当且仅当$x=0$时, $f(x)$取得最大值,最大值是$0$.

item[(2)] extbf{证法一.}
[g(a)+g(b)-2 gleft(frac{a+b}{2} ight)=a ln a+b ln b-(a+b) ln frac{a+b}{2}=a ln frac{2 a}{a+b}+b ln frac{2 b}{a+b}.]
由(1)的结论知$ln (1+x)-x<0$ ($x>-1$且$x eq 0$),由题设$0<a<b$,得$frac{b-a}{2 a}>0,-1<frac{a-b}{2 b}<0$,因此
[ln frac{2 a}{a+b}=-ln left(1+frac{b-a}{2 a} ight)>-frac{b-a}{2 a},quad ln frac{2 b}{a+b}=-ln left(1+frac{a-b}{2 b} ight)>-frac{a-b}{2 b}.]
所以
[a ln frac{2 a}{a+b}+b ln frac{2 b}{a+b}>-frac{b-a}{2}-frac{a-b}{2}=0.]

[frac{2 a}{a+b}<frac{a+b}{2 b}, quad a ln frac{2 a}{a+b}+b ln frac{2 b}{a+b}<a ln frac{a+b}{2 b}+b ln frac{2 b}{a+b}=(b-a) ln frac{2 b}{a+b}<(b-a) ln 2.]
综上, $0<g(a)+g(b)-2 gleft(frac{a+b}{2} ight)<(b-a) ln 2$.


extbf{证法二.} $g(x)=xln x,g'(x)=ln x+1$,设$F(x)=g(a)+g(x)-2 gleft(frac{a+x}{2} ight)$,则$F^{prime}(x)=g^{prime}(x)-2left[gleft(frac{a+x}{2} ight) ight]^{prime}=ln x-ln frac{a+x}{2}$.当$0<x<a$时$F'(x)<0$,因此$F(x)$在$(0,a)$内为减函数;当$x>a$时$F'(x)>0$,因此$F(x)$在$(a,+infty)$上为增函数.从而,当$x=a$时, $F(x)$有极小值$F(a)$.因为$F(a)=0,b>a$,所以$F(b)>0$,即$0<g(a)+g(b)-2 gleft(frac{a+b}{2} ight)$.

设$G(x)=F(x)-(x-a) ln 2$,则$G^{prime}(x)=ln x-ln frac{a+x}{2}-ln 2=ln x-ln (a+x)$.当$x>0$时$G'(x)<0$,因此$G(x)$在$(0,+infty)$上为减函数.因为$G(a)=0,b>a$,所以$G(b)<0$,即$g(a)+g(b)-2 gleft(frac{a+b}{2} ight)<(b-a) ln 2$.
end{enumerate}
end{solution}

egin{problem}


end{problem}
egin{solution}

end{solution}

原文地址:https://www.cnblogs.com/Eufisky/p/12714759.html